ADDITIVE NUMBER THEORY: NOTES AND SOME PRO-
BLEMS
B R Shankar
Department of Mathematical and Computational Sciences, National Institute of Technology Karnataka, Surathkal,
Mangalore - 575025 (India).
E-mail:shankarbr@nitk.edu.in
ORCID:0000-0001-5084-0567
Reception: 24/10/2022 Acceptance: 08/11/2022 Publication: 29/12/2022
Suggested citation:
B.R. Shankar (2022). Additive Number Theory: Notes and Some Problems. 3C Empresa. Investigación y pensamiento
crítico,11 (2), 186-196.https://doi.org/10.17993/3cemp.2022.110250.186-196
https://doi.org/10.17993/3cemp.2022.110250.186-196
ABSTRACT
A brief account of some of the major results in additive number theory is given along with a small list
of problems.
KEYWORDS
Number theory
https://doi.org/10.17993/3cemp.2022.110250.186-196
3C Empresa. Investigación y pensamiento crítico. ISSN: 2254-3376
Ed. 50 Vol. 11 N.º 2 August - December 2022
186
ADDITIVE NUMBER THEORY: NOTES AND SOME PRO-
BLEMS
B R Shankar
Department of Mathematical and Computational Sciences, National Institute of Technology Karnataka, Surathkal,
Mangalore - 575025 (India).
E-mail:shankarbr@nitk.edu.in
ORCID:0000-0001-5084-0567
Reception: 24/10/2022 Acceptance: 08/11/2022 Publication: 29/12/2022
Suggested citation:
B.R. Shankar (2022). Additive Number Theory: Notes and Some Problems. 3C Empresa. Investigación y pensamiento
crítico,11 (2), 186-196.https://doi.org/10.17993/3cemp.2022.110250.186-196
https://doi.org/10.17993/3cemp.2022.110250.186-196
ABSTRACT
A brief account of some of the major results in additive number theory is given along with a small list
of problems.
KEYWORDS
Number theory
https://doi.org/10.17993/3cemp.2022.110250.186-196
187
3C Empresa. Investigación y pensamiento crítico. ISSN: 2254-3376
Ed. 50 Vol. 11 N.º 2 August - December 2022
1 INTRODUCTION
Additive number theory is a relatively young discipline and has seen some spectacular progress in the
last few decades. The aim of this short article is to give a brief description of some of the major results
and also list a few problems that are easy to state and undertstand but not clear how they can (or will)
be solved. The goal is to stimulate and kindle interest in the subject. This is a review paper and most
of the material is well known. The papers of Melvyn B. Nathanson [3], [4], Kannan Soundararajan [5],
Andrew Granville [2], and H V Chu et al [1] are especially informative and detailed. We begin with a
well known and popular result, the Friendship theorem. Friendship Theorem
At any party with at least six people, there are three people who are all either mutual acquaintances
(each one knows the other two) or mutual strangers.
Ramsey Theory
For any given integer
c
, any given integers
n1,...,n
c
, there is a number,
R
(
n1,...,n
c
), such that if the
edges of a complete graph of order
R
(
n1,...,n
c
)are coloured with
c
different colours, then for some
i
between 1and c, it must contain a complete subgraph of order niwhose edges are all colour i.
The special case above has c=2and n1=n2=3.
Broad Philosophy
Can find order in disorder. Are there such analogs in Number theory? Answer is
YES. Order is Arithmetic progressions (APs). 3-AP means 3 consecutive terms in AP and k-AP means
k consecutive terms in AP. One can ask under what conditions will a set A of integers contain atleast
one 3-AP? No 3-AP at all? Infinitely many 3-APs?
Broadly, given a sufficiently large set of integers
A
we interested in understanding additive patterns
that appear in
A
. An important example is whether
A
contains non-trivial arithmetic progressions of
some given length k.
Reason
: They are quite indestructible structures. They are preserved under translations and dilations
of A, and they cannot be excluded for trivial congruence reasons.
For example the pattern
a
,
b
and
a
+
b
all being in the set seems quite close to the arithmetic progression
case
a, b,
(
a
+
b
)
/
2, but the former case can never occur in any subset of the odd integers (and such
subsets can be very large).
Other questions
: Whether all numbers can be written as a sum of
s
elements from a given set
A
. For
example, all numbers are sums of four squares, nine cubes etc. Waring’s problem and the Goldbach
conjectures are two classical examples. In the same spirit, given a set
A
of
N
integers we may ask for
information about the sumset
A
+
A
:=
{a
+
b
:
a, b A}
. If there are not too many coincidences, then
we may expect
|A
+
A|N2
. But when
A
is an AP,
|A
+
A|≤
2
|A|−
1. The subject may be said to
begin with a beautiful result of van der Waerden (1927).
Theorem 1. (van der Waerden)
. Let
k
and
r
be given. There exists a number
N
=
N
(
k, r
)such
that if the integers in [1
,N
]are colored using
r
colors, then there is a non-trivial monochromatic k term
arithmetic progression.
van der Waerden’s proof was by an ingenious elementary induction argument on
k
and
r
. The proof
does not give any good bound on how large
N
(
k, r
)needs to be. A more general result was subsequently
found by Hales and Jewett (1963), with a nice refinement of Shelah (1988), but again the bounds for
the van der Waerden numbers are quite poor.
Theorem 2. (Schur)
. Given any positive integer
r>
1, if
NN
(
r
)and the integers in [1
,N
]are
colored using rcolors then there is a monochromatic solution to x+y=z.
Lemma 1.
Suppose that the edges of the complete graph
KN
are colored using
r
colors. If
NN
(
r
)
then there is a monochromatic triangle.
Proof:
We will use induction on
r
. It is very well known that if
r
=2and
N
6then there is a
monochromatic triangle. Suppose we know the result for
r
1colorings, and we need
NN
(
r
1)
for that result. Pick a vertex. There are
N
1edges coming out of it. So for some color there are
[(
N
1)
/r
]+1edges starting from this vertex having the same color. Now the complete graph on the
https://doi.org/10.17993/3cemp.2022.110250.186-196
other vertices of these edges must be colored using only
r
1colors. Thus if
NrN
(
r
1)
r
+2we
are done.
Proof of Schur’s Theorem:
Consider the complete graph on
N
vertices labeled 1through
N
.
Color the edge joining
a
to
b
using the color of
|ab|
. By our lemma, if
N
is large then there is a
monochromatic triangle. Suppose its vertices are
a<b<c
, then (
ca
)=(
cb
)+(
ba
)is a solution,
proving Schur’s theorem.
Theorem 3. (Hales-Jewett)
. Let
k
and
r
be given. There exists a number
N
=
N
(
k, r
)such that if
the points in [1
,k
]
N
are colored using
r
colors then there is a monochromatic “combinatorial line”. Here
a combinatorial line is a collection of
k
points of the following type: certain of the coordinates are fixed,
and a certain non-empty set of coordinates are designated as “wildcards” taking all the values from 1 to
k.
A picturesque way of describing the Hales-Jewett theorem is that a “tic-tac-toe” game of getting k in
a row, played by r players, always has a result in sufficiently high dimensions. Since there is obviously
no disadvantage to going first, the first player wins; but no constructive strategy solving the game
is known. One can recover van der Waerden’s theorem by thinking of [1
,k
]
N
as giving the base
k
digits(shifted by 1) of numbers in [0,k
N1].
Erdos and Turan proposed a stronger form of the van der Waerden, partly in the hope that the
solution to the stronger problem would lead to a better version of van der Waerden’s theorem.
The Erdos-Turan conjecture
: Let
δ
and
k
be given. There is a number
N
=
N
(
k, δ
)such that any
set A[1,N]with |A|≥δN contains a non-trivial arithmetic progression of lenghth k.
In 1953, Roth proved the Erdos - Turan conjecture in the case k=3.
Theorem 4. (Roth)
. There exists a positive constant
C
such that if
A
[1
,N
]with
|A|≥CN/ log log N
then
A
has a non-trivial three term AP. In other words,
N
(
δ,
3)
exp
(
exp
(
C
)) for some positive
constant C.
This stronger result does in fact give a good bound on the van der Waerden numbers for
k
=3. We
know now(Bourgain), that
|A|N
(
log log N/log N
)
1/2
suffices. Thus the double exponential bound
can be replaced by a single exponential.
Let
r3
(
N
)denote the size of the largest subset of [1
,N
]having no non-trivial three term APs. Then
as mentioned above,
r3
(
N
)
Nlog log N/log N
. What is the true nature of
r3
(
N
)?Ifwepicka
random set
A
in [1
,N
]we may expect that it has about
|A|3/N
three term APs. This suggests that
r3
(
N
)is perhaps of size
N1
3
. However, in 1946 Behrend found an ingenious construction that does
much much better.
Theorem 5. (Behrend)
. There exists a set
A
[1
,N
]with
|A|Bexp
(
clogN
)containing no
non-trivial three term arithmetic progressions. In other words r3(N)Nexp(clog N).
Roth’s proof is based on Fourier analysis. It falls naturally into two parts: either the set
A
looks
random in which case we may easily count the number of three term progressions, or the set has some
structure which can be exploited to find a subset with increased density. The crucial point is that the
idea of randomness here can be made precise in terms of the size of the Fourier coefficients of the set.
This argument is quite hard to generalize to four term progressions (or longer), and was only extended
recently with the spectacular work of Gowers.
Returning to the Erdős-Turan conjecture, the next big breakthrough was made by Szemeredi who
in 1969 established the case
k
=4, and in 1975 dealt with the general case
k
5. His proof was a
tour-de-force of extremely ingenious and difficult combinatorics. One of his ingredients was van der
Waerden’s theorem, and so this did not lead to a good bound there.
Theorem 6. (Szemeredi)
. Given
k
and
δ>
0, there exists
N
=
N
(
k, δ
)such that any set
A
[1
,N
]
with |A|≥δN contains a non-trivial kterm arithmetic progression.
https://doi.org/10.17993/3cemp.2022.110250.186-196
3C Empresa. Investigación y pensamiento crítico. ISSN: 2254-3376
Ed. 50 Vol. 11 N.º 2 August - December 2022
188
1 INTRODUCTION
Additive number theory is a relatively young discipline and has seen some spectacular progress in the
last few decades. The aim of this short article is to give a brief description of some of the major results
and also list a few problems that are easy to state and undertstand but not clear how they can (or will)
be solved. The goal is to stimulate and kindle interest in the subject. This is a review paper and most
of the material is well known. The papers of Melvyn B. Nathanson [3], [4], Kannan Soundararajan [5],
Andrew Granville [2], and H V Chu et al [1] are especially informative and detailed. We begin with a
well known and popular result, the Friendship theorem. Friendship Theorem
At any party with at least six people, there are three people who are all either mutual acquaintances
(each one knows the other two) or mutual strangers.
Ramsey Theory
For any given integer
c
, any given integers
n1,...,n
c
, there is a number,
R
(
n1,...,n
c
), such that if the
edges of a complete graph of order
R
(
n1,...,n
c
)are coloured with
c
different colours, then for some
i
between 1and c, it must contain a complete subgraph of order niwhose edges are all colour i.
The special case above has c=2and n1=n2=3.
Broad Philosophy
Can find order in disorder. Are there such analogs in Number theory? Answer is
YES. Order is Arithmetic progressions (APs). 3-AP means 3 consecutive terms in AP and k-AP means
k consecutive terms in AP. One can ask under what conditions will a set A of integers contain atleast
one 3-AP? No 3-AP at all? Infinitely many 3-APs?
Broadly, given a sufficiently large set of integers
A
we interested in understanding additive patterns
that appear in
A
. An important example is whether
A
contains non-trivial arithmetic progressions of
some given length k.
Reason
: They are quite indestructible structures. They are preserved under translations and dilations
of A, and they cannot be excluded for trivial congruence reasons.
For example the pattern
a
,
b
and
a
+
b
all being in the set seems quite close to the arithmetic progression
case
a, b,
(
a
+
b
)
/
2, but the former case can never occur in any subset of the odd integers (and such
subsets can be very large).
Other questions
: Whether all numbers can be written as a sum of
s
elements from a given set
A
. For
example, all numbers are sums of four squares, nine cubes etc. Waring’s problem and the Goldbach
conjectures are two classical examples. In the same spirit, given a set
A
of
N
integers we may ask for
information about the sumset
A
+
A
:=
{a
+
b
:
a, b A}
. If there are not too many coincidences, then
we may expect
|A
+
A|N2
. But when
A
is an AP,
|A
+
A|≤
2
|A|−
1. The subject may be said to
begin with a beautiful result of van der Waerden (1927).
Theorem 1. (van der Waerden)
. Let
k
and
r
be given. There exists a number
N
=
N
(
k, r
)such
that if the integers in [1
,N
]are colored using
r
colors, then there is a non-trivial monochromatic k term
arithmetic progression.
van der Waerden’s proof was by an ingenious elementary induction argument on
k
and
r
. The proof
does not give any good bound on how large
N
(
k, r
)needs to be. A more general result was subsequently
found by Hales and Jewett (1963), with a nice refinement of Shelah (1988), but again the bounds for
the van der Waerden numbers are quite poor.
Theorem 2. (Schur)
. Given any positive integer
r>
1, if
NN
(
r
)and the integers in [1
,N
]are
colored using rcolors then there is a monochromatic solution to x+y=z.
Lemma 1.
Suppose that the edges of the complete graph
KN
are colored using
r
colors. If
NN
(
r
)
then there is a monochromatic triangle.
Proof:
We will use induction on
r
. It is very well known that if
r
=2and
N
6then there is a
monochromatic triangle. Suppose we know the result for
r
1colorings, and we need
NN
(
r
1)
for that result. Pick a vertex. There are
N
1edges coming out of it. So for some color there are
[(
N
1)
/r
]+1edges starting from this vertex having the same color. Now the complete graph on the
https://doi.org/10.17993/3cemp.2022.110250.186-196
other vertices of these edges must be colored using only
r
1colors. Thus if
NrN
(
r
1)
r
+2we
are done.
Proof of Schur’s Theorem:
Consider the complete graph on
N
vertices labeled 1through
N
.
Color the edge joining
a
to
b
using the color of
|ab|
. By our lemma, if
N
is large then there is a
monochromatic triangle. Suppose its vertices are
a<b<c
, then (
ca
)=(
cb
)+(
ba
)is a solution,
proving Schur’s theorem.
Theorem 3. (Hales-Jewett)
. Let
k
and
r
be given. There exists a number
N
=
N
(
k, r
)such that if
the points in [1
,k
]
N
are colored using
r
colors then there is a monochromatic “combinatorial line”. Here
a combinatorial line is a collection of
k
points of the following type: certain of the coordinates are fixed,
and a certain non-empty set of coordinates are designated as “wildcards” taking all the values from 1 to
k.
A picturesque way of describing the Hales-Jewett theorem is that a “tic-tac-toe” game of getting k in
a row, played by r players, always has a result in sufficiently high dimensions. Since there is obviously
no disadvantage to going first, the first player wins; but no constructive strategy solving the game
is known. One can recover van der Waerden’s theorem by thinking of [1
,k
]
N
as giving the base
k
digits(shifted by 1) of numbers in [0,k
N1].
Erdos and Turan proposed a stronger form of the van der Waerden, partly in the hope that the
solution to the stronger problem would lead to a better version of van der Waerden’s theorem.
The Erdos-Turan conjecture
: Let
δ
and
k
be given. There is a number
N
=
N
(
k, δ
)such that any
set A[1,N]with |A|≥δN contains a non-trivial arithmetic progression of lenghth k.
In 1953, Roth proved the Erdos - Turan conjecture in the case k=3.
Theorem 4. (Roth)
. There exists a positive constant
C
such that if
A
[1
,N
]with
|A|≥CN/ log log N
then
A
has a non-trivial three term AP. In other words,
N
(
δ,
3)
exp
(
exp
(
C
)) for some positive
constant C.
This stronger result does in fact give a good bound on the van der Waerden numbers for
k
=3. We
know now(Bourgain), that
|A|N
(
log log N/log N
)
1/2
suffices. Thus the double exponential bound
can be replaced by a single exponential.
Let
r3
(
N
)denote the size of the largest subset of [1
,N
]having no non-trivial three term APs. Then
as mentioned above,
r3
(
N
)
Nlog log N/log N
. What is the true nature of
r3
(
N
)?Ifwepicka
random set
A
in [1
,N
]we may expect that it has about
|A|3/N
three term APs. This suggests that
r3
(
N
)is perhaps of size
N1
3
. However, in 1946 Behrend found an ingenious construction that does
much much better.
Theorem 5. (Behrend)
. There exists a set
A
[1
,N
]with
|A|Bexp
(
clogN
)containing no
non-trivial three term arithmetic progressions. In other words r3(N)Nexp(clog N).
Roth’s proof is based on Fourier analysis. It falls naturally into two parts: either the set
A
looks
random in which case we may easily count the number of three term progressions, or the set has some
structure which can be exploited to find a subset with increased density. The crucial point is that the
idea of randomness here can be made precise in terms of the size of the Fourier coefficients of the set.
This argument is quite hard to generalize to four term progressions (or longer), and was only extended
recently with the spectacular work of Gowers.
Returning to the Erdős-Turan conjecture, the next big breakthrough was made by Szemeredi who
in 1969 established the case
k
=4, and in 1975 dealt with the general case
k
5. His proof was a
tour-de-force of extremely ingenious and difficult combinatorics. One of his ingredients was van der
Waerden’s theorem, and so this did not lead to a good bound there.
Theorem 6. (Szemeredi)
. Given
k
and
δ>
0, there exists
N
=
N
(
k, δ
)such that any set
A
[1
,N
]
with |A|≥δN contains a non-trivial kterm arithmetic progression.
https://doi.org/10.17993/3cemp.2022.110250.186-196
189
3C Empresa. Investigación y pensamiento crítico. ISSN: 2254-3376
Ed. 50 Vol. 11 N.º 2 August - December 2022
An entirely different approach was opened by the work of Furstenberg (1977) who used ergodic
theoretic methods to obtain a new proof of Szemeredi’s theorem. The ergodic theoretic approach also
did not lead to any good bounds, but was useful in proving other results previously inaccessible. For
example, it led to a multi-dimensional version of Szemeredi’s theorem, also a density version of the
Hales-Jewett theorem (due to Katznelson and Ornstein), and also allowed for the common difference of
the APs to have special shapes (e.g. squares).
In 1998-2001 Gowers made a major breakthrough by extending Roth’s harmonic analysis techniques
to prove Szemeredi’s theorem. This approach finally gave good bounds for the van der Waerden numbers.
Theorem 7. (Gowers)
. There exists a positive constant
ck
such that any subset
A
in [1
,N
]with
|A|N/(log log N)ckcontains a non-trivial kterm arithmetic progression.
One of the major insights of Gowers is the development of a “quadratic theory of Fourier analysis”
which substitutes for the “linear Fourier analysis” used in Roth’s theorem. Gowers’s ideas have trans-
formed the field, opening the door to many spectacular results, most notably the work of Green and
Tao.
The Green-Tao Theorem (2003)
. The primes contain arbitrarily long non-trivial arithmetic
progressions.
By the Prime Number Theorem, upto
N
there are about
N/ log N
primes. This density is much
smaller than what would be covered by Gowers theorem; even in the case
k
=3it is not covered by the
best known results on r3(N). Another result is the celebrated three primes theorem.
Theorem 8. (Vinogradov, 1937). Every large odd number is the sum of three primes.
Another brilliant result of Green and Tao, developing Gowers ideas, is that
r4
(
N
)
N
(
log N
)
c
where r4(N)denotes the largest cardinality of a set in [1,N]containing no four term progressions.
Another theme is Freiman’s theorem on sumsets. If
A
is a set of
N
integers then
A
+
A
is bounded above
by
N
(
N
+ 1)
/
2, and below by 2
N
1. The lower bound is attained only when
A
is highly structured,
and is an arithmetic progression of length
N
. Clearly if
A
is a subset of an arithmetic progression of
length
CN
then
|A
+
A|≤
2
C|A|
. More generally suppose
d1, ..., dk
are given numbers, and consider
the set
{a0
+
a1d1
+
...
+
akdk
:1
aiNi
for 1
ik}
. We may think of this as a generalized
arithmetic progression (GAP) of dimension
k
. Note that this GAP has cardinality at most
N1...Nk
. If
these sums are all distinct (so that the cardinality equals
N1...Nk
) we call the GAP
proper
. Note that
if A is contained in a GAP of dimension
k
and size
CN
then
|A
+
A|≤
2
kCN
. Freiman’s theorem
provides a converse to this showing that all sets with small sumsets must arise in this fashion.
Theorem 9. (Freiman)
. If
A
is a set with
|A
+
A|≤C|A|
then there exists a proper GAP of dimension
k(bounded in terms of C) and size C1|A|for some constant C1depending only on C.
Qualitatively Freiman’s theorem says that any set with a small sumset looks like an arithmetic
progression. Similarly we may expect that a set with a small product set should look like a geometric
progression. But of course no set looks simultaneously like an arithmetic and a geometric progression!
This led to the following conjecture which says either the sumset or the product set must be large.
Erdős-Szemeredi Conjecture.
If
A
is a set of
N
integers then
|A
+
A|
+
|A.A|N2
, for any
>0.
This is currently known for
>
3
/
4The sum-product theory (and its generalizations) is another
very active problem in additive combinatorics, and has led to many important applications (bounding
exponential sums etc).
Poincare recurrence:
Let
X
be a probability space with measure
µ
, and let
T
be a measure preserving
transformation (so
µ
(
T1A
)=
µ
(
A
)). For any set
V
with positive measure there exists a point
xV
such that for some natural number n,Tnxalso is in V.
https://doi.org/10.17993/3cemp.2022.110250.186-196
Proof:
This is very simple: note that the sets
V,T1V,T2V, ...
cannot all be disjoint. Therefore
TmVTmnV
=
φ
for some natural numbers
m
and
n
. But this gives readily that
VTnV
=
φ
as
needed.
It is clear from the proof that the number
n
in Poincare’s result may be found below 1
(
V
). As an
example, we may take
X
to be the circle
R/Z
, and take
V
to be the interval [
1
/
2
Q,
1
/
2
Q
], and
T
to
be the map xx+θfor some fixed number θ. We thus obtain:
Theorem 10. (Dirichlet)
. For any real number
θ
, and any
Q
1there exists 1
qQ
such that
|||| 1/Q. Here ||x|| denotes the distance between xand its nearest integer.
If
X
happens also to be a separable (covered by countably many open sets) metric space, then we
can divide
X
into countably many balls of radius
/
2. Then it follows that almost every point of
X
returns to within of itself. That is, almost every point is recurrent.
We don’t really need a probability space to find recurrence. Birkhoff realized that this can be
achieved purely topologically and holds for compact metric spaces.
Theorem 11. (Birkhoff’s Recurrence)
. Let
X
be a compact metric space, and
T
be a continuous
map. Then there exists a recurrent point in
X
; namely, a point
x
such that there is a sequence
nk→∞
with Tnkxx.
Proof:
Since
X
is compact, any nested sequence of non-empty closed sets
Y1Y2Y3...
has a
non-empty intersection. Consider
T
-invariant closed subsets of
X
; that is,
Y
with
TY Y
. By Zorn’s
lemma and our observation above, there exists a non-empty minimal closed invariant set
Y
. Let
y
be
any point in
Y
and consider the closure of
y, Ty, T2y, ...
This set is plainly a closed invariant subset of
Y, and by minimality equals Y. Therefore yis recurrent.
These are some basic simple results, of the same depth as Dirichlet’s pigeonhole principle and its
application to Diophantine approximation. In the example of Diophantine approximation, we see that
if |||| is small then so are ||2||,||3|| etc. This suggests the possibility of multiple recurrence.
Topological Multiple Recurrence.
Let
X
be a compact metric space, and
T
be a continuous map.
For any integer
k
1there exists a point
xX
and a sequence
nl→∞
with
Tjnlxx
for each
1jk.
This theorem is analogous to van der Waerden’s theorem, and indeed implies it. To see this, let
Λ=
{
1
, ..., r}
represent
r
colors, and consider Ω=Λ
Z.
Thus is the space of all
r
colorings of the
integers, and by
x
we understand a particular
r
coloring of the integers. We make into a compact
metric space (check using sequential compactness), by taking as the metric
d
(
x, y
)=0if
x
=
y
and
d
(
x, y
)=2
l
where
l
is the least magnitude for which either
x
(
l
)
=
y
(
l
)or
x
(
l
)
=
y
(
l
). We define
the shift map Tby Tx(n)=x(n+ 1).
Now suppose we are given a coloring
ξ
of the integers. Take
X
to be the closure of
Tnξ
where
n
ranges
over all integers. By definition this is a closed invariant compact metric space, and so by the Topological
Multiple Recurrence Theorem there is a
xX
and some
nZ
with
x
(0) =
x
(
n
)=
x
(2
n
)=
...
=
x
(
kn
).
But from the definition of the space
X
we may find an
mZ
such that
Tmξ
and
x
agree on the
interval [
kn, kn
]. Then it follows that
ξ
(
m
)=
ξ
(
m
+
n
)=
...
=
ξ
(
m
+
kn
)producing a
k
+1term AP.
The above argument gives an infinitary version of the van der Waerden theorem where we color all
the integers. But from it we may deduce the finite version. Suppose not, and there are
r
colorings of
[
N,N
]with no monochromatic k-APs for each natural number
N
. Extend each of these colorings
arbitrarily to
Z
, obtaining an element in . By compactness we may find a limit point in of these
elements. That limit point defines a coloring of
Z
containing no monochromatic k-APs, and this is a
contradiction.
The ergodic theoretic analog of Szemeredi’s theorem is Furstenberg’s multiple recurrence theorem
for measure preserving transformations, and this implies Szemeredi by an argument similar to the one
above.
https://doi.org/10.17993/3cemp.2022.110250.186-196
3C Empresa. Investigación y pensamiento crítico. ISSN: 2254-3376
Ed. 50 Vol. 11 N.º 2 August - December 2022
190
An entirely different approach was opened by the work of Furstenberg (1977) who used ergodic
theoretic methods to obtain a new proof of Szemeredi’s theorem. The ergodic theoretic approach also
did not lead to any good bounds, but was useful in proving other results previously inaccessible. For
example, it led to a multi-dimensional version of Szemeredi’s theorem, also a density version of the
Hales-Jewett theorem (due to Katznelson and Ornstein), and also allowed for the common difference of
the APs to have special shapes (e.g. squares).
In 1998-2001 Gowers made a major breakthrough by extending Roth’s harmonic analysis techniques
to prove Szemeredi’s theorem. This approach finally gave good bounds for the van der Waerden numbers.
Theorem 7. (Gowers)
. There exists a positive constant
ck
such that any subset
A
in [1
,N
]with
|A|N/(log log N)ckcontains a non-trivial kterm arithmetic progression.
One of the major insights of Gowers is the development of a “quadratic theory of Fourier analysis”
which substitutes for the “linear Fourier analysis” used in Roth’s theorem. Gowers’s ideas have trans-
formed the field, opening the door to many spectacular results, most notably the work of Green and
Tao.
The Green-Tao Theorem (2003)
. The primes contain arbitrarily long non-trivial arithmetic
progressions.
By the Prime Number Theorem, upto
N
there are about
N/ log N
primes. This density is much
smaller than what would be covered by Gowers theorem; even in the case
k
=3it is not covered by the
best known results on r3(N). Another result is the celebrated three primes theorem.
Theorem 8. (Vinogradov, 1937). Every large odd number is the sum of three primes.
Another brilliant result of Green and Tao, developing Gowers ideas, is that
r4
(
N
)
N
(
log N
)
c
where r4(N)denotes the largest cardinality of a set in [1,N]containing no four term progressions.
Another theme is Freiman’s theorem on sumsets. If
A
is a set of
N
integers then
A
+
A
is bounded above
by
N
(
N
+ 1)
/
2, and below by 2
N
1. The lower bound is attained only when
A
is highly structured,
and is an arithmetic progression of length
N
. Clearly if
A
is a subset of an arithmetic progression of
length
CN
then
|A
+
A|≤
2
C|A|
. More generally suppose
d1, ..., dk
are given numbers, and consider
the set
{a0
+
a1d1
+
...
+
akdk
:1
aiNi
for 1
ik}
. We may think of this as a generalized
arithmetic progression (GAP) of dimension
k
. Note that this GAP has cardinality at most
N1...Nk
. If
these sums are all distinct (so that the cardinality equals
N1...Nk
) we call the GAP
proper
. Note that
if A is contained in a GAP of dimension
k
and size
CN
then
|A
+
A|≤
2
kCN
. Freiman’s theorem
provides a converse to this showing that all sets with small sumsets must arise in this fashion.
Theorem 9. (Freiman)
. If
A
is a set with
|A
+
A|≤C|A|
then there exists a proper GAP of dimension
k(bounded in terms of C) and size C1|A|for some constant C1depending only on C.
Qualitatively Freiman’s theorem says that any set with a small sumset looks like an arithmetic
progression. Similarly we may expect that a set with a small product set should look like a geometric
progression. But of course no set looks simultaneously like an arithmetic and a geometric progression!
This led to the following conjecture which says either the sumset or the product set must be large.
Erdős-Szemeredi Conjecture.
If
A
is a set of
N
integers then
|A
+
A|
+
|A.A|N2
, for any
>0.
This is currently known for
>
3
/
4The sum-product theory (and its generalizations) is another
very active problem in additive combinatorics, and has led to many important applications (bounding
exponential sums etc).
Poincare recurrence:
Let
X
be a probability space with measure
µ
, and let
T
be a measure preserving
transformation (so
µ
(
T1A
)=
µ
(
A
)). For any set
V
with positive measure there exists a point
xV
such that for some natural number n,Tnxalso is in V.
https://doi.org/10.17993/3cemp.2022.110250.186-196
Proof:
This is very simple: note that the sets
V,T1V,T2V, ...
cannot all be disjoint. Therefore
TmVTmnV
=
φ
for some natural numbers
m
and
n
. But this gives readily that
VTnV
=
φ
as
needed.
It is clear from the proof that the number
n
in Poincare’s result may be found below 1
(
V
). As an
example, we may take
X
to be the circle
R/Z
, and take
V
to be the interval [
1
/
2
Q,
1
/
2
Q
], and
T
to
be the map xx+θfor some fixed number θ. We thus obtain:
Theorem 10. (Dirichlet)
. For any real number
θ
, and any
Q
1there exists 1
qQ
such that
|||| 1/Q. Here ||x|| denotes the distance between xand its nearest integer.
If
X
happens also to be a separable (covered by countably many open sets) metric space, then we
can divide
X
into countably many balls of radius
/
2. Then it follows that almost every point of
X
returns to within of itself. That is, almost every point is recurrent.
We don’t really need a probability space to find recurrence. Birkhoff realized that this can be
achieved purely topologically and holds for compact metric spaces.
Theorem 11. (Birkhoff’s Recurrence)
. Let
X
be a compact metric space, and
T
be a continuous
map. Then there exists a recurrent point in
X
; namely, a point
x
such that there is a sequence
nk→∞
with Tnkxx.
Proof:
Since
X
is compact, any nested sequence of non-empty closed sets
Y1Y2Y3...
has a
non-empty intersection. Consider
T
-invariant closed subsets of
X
; that is,
Y
with
TY Y
. By Zorn’s
lemma and our observation above, there exists a non-empty minimal closed invariant set
Y
. Let
y
be
any point in
Y
and consider the closure of
y, Ty, T2y, ...
This set is plainly a closed invariant subset of
Y, and by minimality equals Y. Therefore yis recurrent.
These are some basic simple results, of the same depth as Dirichlet’s pigeonhole principle and its
application to Diophantine approximation. In the example of Diophantine approximation, we see that
if |||| is small then so are ||2||,||3|| etc. This suggests the possibility of multiple recurrence.
Topological Multiple Recurrence.
Let
X
be a compact metric space, and
T
be a continuous map.
For any integer
k
1there exists a point
xX
and a sequence
nl→∞
with
Tjnlxx
for each
1jk.
This theorem is analogous to van der Waerden’s theorem, and indeed implies it. To see this, let
Λ=
{
1
, ..., r}
represent
r
colors, and consider Ω=Λ
Z.
Thus is the space of all
r
colorings of the
integers, and by
x
we understand a particular
r
coloring of the integers. We make into a compact
metric space (check using sequential compactness), by taking as the metric
d
(
x, y
)=0if
x
=
y
and
d
(
x, y
)=2
l
where
l
is the least magnitude for which either
x
(
l
)
=
y
(
l
)or
x
(
l
)
=
y
(
l
). We define
the shift map Tby Tx(n)=x(n+ 1).
Now suppose we are given a coloring
ξ
of the integers. Take
X
to be the closure of
Tnξ
where
n
ranges
over all integers. By definition this is a closed invariant compact metric space, and so by the Topological
Multiple Recurrence Theorem there is a
xX
and some
nZ
with
x
(0) =
x
(
n
)=
x
(2
n
)=
...
=
x
(
kn
).
But from the definition of the space
X
we may find an
mZ
such that
Tmξ
and
x
agree on the
interval [
kn, kn
]. Then it follows that
ξ
(
m
)=
ξ
(
m
+
n
)=
...
=
ξ
(
m
+
kn
)producing a
k
+1term AP.
The above argument gives an infinitary version of the van der Waerden theorem where we color all
the integers. But from it we may deduce the finite version. Suppose not, and there are
r
colorings of
[
N,N
]with no monochromatic k-APs for each natural number
N
. Extend each of these colorings
arbitrarily to
Z
, obtaining an element in . By compactness we may find a limit point in of these
elements. That limit point defines a coloring of
Z
containing no monochromatic k-APs, and this is a
contradiction.
The ergodic theoretic analog of Szemeredi’s theorem is Furstenberg’s multiple recurrence theorem
for measure preserving transformations, and this implies Szemeredi by an argument similar to the one
above.
https://doi.org/10.17993/3cemp.2022.110250.186-196
191
3C Empresa. Investigación y pensamiento crítico. ISSN: 2254-3376
Ed. 50 Vol. 11 N.º 2 August - December 2022
Theorem 12. (Furstenberg)
. Let
X
be a probability measure space and let
T
be a measure preserving
transformation. If
V
is a set of positive measure, then there exists a natural number
n
such that
VTnVT2nV... TknVhas positive measure.
Behrend’s Example. Behrend constructed a surprisingly large set in [1,N]with no 3-APs.
Behrend’s Theorem A.
There is a set
A
in [1
,N
]which is free of 3 APs and satisfies
|A|
Nexp(clog N). Here cis an absolute positive constant.
Behrend’s Theorem B.
There exists a set
A
in [1
,N
]with
|A|≥δN
which has
δclog (1)N2
three
term progressions. Here cis an absolute positive constant, and δ>0.
Theorem 13. (Varnavides)
. For every
δ>
0there exists
C
(
δ
)
>
0such that if
A
[1
,N
]with
|A|≥δN then Acontains at least C(δ)N2three term progressions.
Erdos and Szemeredi
: Expect that additive and multiplicative structures are independent. Hence
one of the two sets, i.e., sumset and product set, must be large.
Theorem 14. (Solymosi)
. Let
A
,
B
and
C
be finite sets of real numbers, each having at least two
elements. Then
|A+B|×|A.C|(|A|3|B||C|)1/2
In particular, if
A, B
and
C
all have cardinality
N
then either
A
+
B
or
A.C
has cardinality
N5/4
.
Note:
Both,
Erdos-Turan conjecture
and
Szemeredi’s Theorem
assume that the set
A
has a
positive density (Schnirelman density). However, the primes have zero density because of the
prime
number theorem. Hence Green - Tao theorem is much harder.
Now we consider finite sets: For any set Aof integers, we define the sumset
A+A={a+a:a, aA}
and the difference set
AA={aa:a, aA}.
We consider finite sets of integers, and the relative sizes of their sumsets and difference sets. If
A
is a
finite set of integers and
x, y Z
, then the
translation
of
A
by
x
is the set
x
+
A
=
{x
+
a
:
aA}
and the dilation of Aby yis yA={ya :aA}. We have
(x+A)+(x+A)=2x+2A
and
(x+A)(x+A)=AA.
Similarly,
yA+yA=y(A+A)
and
yAyA=y(AA).
It follows that
|(x+yA)+(x+yA)|=|2A|
and
|(x+yA)(x+yA)|=|AA|
https://doi.org/10.17993/3cemp.2022.110250.186-196
So the cardinalities of the sum and difference sets of a finite set of integers are invariant under affine
transformations of the set. Easy to see that if
|A|
=
N
, then
|A
+
A|
is bounded above by
N
(
N
+ 1)
/
2
and below by 2N1. The latter occurs when Ais an AP or symmetric.
The set
A
is symmetric with respect to the integer
z
if
A
=
zA
or, equivalently, if
aA
if and
only if zaA. For example, the set {4,6,7,9}is symmetric with z= 13. If Ais symmetric, then
A+A=A+(zA)=z+(AA)
and so
|A
+
A|
=
|AA|
. Equality also holds if
A
is an AP. Examples of equality exists even if
A
is
neither symmetric nor an AP.
A
=
{
0
,
1
,
3
,
4
,
5
,
8
}
is neither symmetric nor an AP but
|A
+
A|
=
|AA|
.
If A={a, b, c}with a<b<cand a+c=2b, then |A+A|=6<7=|AA|.
If
A
=
{
0
,
2
,
3
,
4
,
7
}
then
A
+
A
= [0
,
14]
\{
1
,
12
,
13
}
,
AA
=[
7
,
7]
\ {−
6
,
6
}
and
|A
+
A|
= 12
<
13=|AA|.
This is the typical situation. Since 2+7=7+2but 27=72
It is natural to expect that in any finite set of integers there are always at least as many differences
as sums. There had been a conjecture, often ascribed incorrectly to John Conway, that asserted that
|A+A|≤|AA|for every finite set Aof integers.
This conjecture is false, and a counterexample is the set A={0,2,3,4,7,11,12,14}, for which
A+A= [0,28] \{1,20,27}
AA=[14,14] \ 6,±13}and |A+A|= 26 >25=|AA|.
Given the existence of such aberrant sets, we can ask for the smallest one. The set
A
above satisfies
|A|=8.
Problem 1.
What is the smallest such A?
i.e. find min{|A|:AZand |A+A|>|AA|}?
Note:
Hegarty has proved that this minimum is indeed equal to 8and is affinely equivalent to the set
A={0,2,3,4,7,11,12,14}. One may also ask:
Problem 2.
What is the structure of finite sets satisfying |A+A|>|AA|?
If
A
is a finite set of integers and
m
is a sufficiently large positive integer (for example,
m>
2 max({|a|:aA}), then the set
At=t1
i=0
aimi:aiAfor i=0,1,...,t1
has the property that
|At
+
At|
=
|A
+
A|t
and
|AtAt|
=
|AA|t
. This can be seen as follows: the
elements of
At
+
At
can be thought of as having a base-m expansion with ’digits’ coming from the set
A+A.
It follows that if |A+A|>|AA|, then At+At|>|AtAt|and, moreover,
lim
t |At+At
|AtAt|= lim
t |A+A|
|AA|t
=
The sequence of sets
{At}
t=1
is the standard parametrized family of sets with more sums than differences
(called MSTD sets).
Problem 3.
Are there other parametrized families of sets satisfying |A+A|>|AA|?
Even though there exist sets
A
that have more sums than differences, such sets should be rare, and
it must be true with the right way of counting that the vast majority of sets satises
|AA|>|A
+
A|
.
Problem 4.
Let
f
(
n
)denote the number sets
A
[0
,n
1] such that
|AA|<|A
+
A|
, and let
f
(
n, k
)denote the
https://doi.org/10.17993/3cemp.2022.110250.186-196
3C Empresa. Investigación y pensamiento crítico. ISSN: 2254-3376
Ed. 50 Vol. 11 N.º 2 August - December 2022
192
Theorem 12. (Furstenberg)
. Let
X
be a probability measure space and let
T
be a measure preserving
transformation. If
V
is a set of positive measure, then there exists a natural number
n
such that
VTnVT2nV... TknVhas positive measure.
Behrend’s Example. Behrend constructed a surprisingly large set in [1,N]with no 3-APs.
Behrend’s Theorem A.
There is a set
A
in [1
,N
]which is free of 3 APs and satisfies
|A|
Nexp(clog N). Here cis an absolute positive constant.
Behrend’s Theorem B.
There exists a set
A
in [1
,N
]with
|A|≥δN
which has
δclog (1)N2
three
term progressions. Here cis an absolute positive constant, and δ>0.
Theorem 13. (Varnavides)
. For every
δ>
0there exists
C
(
δ
)
>
0such that if
A
[1
,N
]with
|A|≥δN then Acontains at least C(δ)N2three term progressions.
Erdos and Szemeredi
: Expect that additive and multiplicative structures are independent. Hence
one of the two sets, i.e., sumset and product set, must be large.
Theorem 14. (Solymosi)
. Let
A
,
B
and
C
be finite sets of real numbers, each having at least two
elements. Then
|A+B|×|A.C|(|A|3|B||C|)1/2
In particular, if
A, B
and
C
all have cardinality
N
then either
A
+
B
or
A.C
has cardinality
N5/4
.
Note:
Both,
Erdos-Turan conjecture
and
Szemeredi’s Theorem
assume that the set
A
has a
positive density (Schnirelman density). However, the primes have zero density because of the
prime
number theorem. Hence Green - Tao theorem is much harder.
Now we consider finite sets: For any set Aof integers, we define the sumset
A+A={a+a:a, aA}
and the difference set
AA={aa:a, aA}.
We consider finite sets of integers, and the relative sizes of their sumsets and difference sets. If
A
is a
finite set of integers and
x, y Z
, then the
translation
of
A
by
x
is the set
x
+
A
=
{x
+
a
:
aA}
and the dilation of Aby yis yA={ya :aA}. We have
(x+A)+(x+A)=2x+2A
and
(x+A)(x+A)=AA.
Similarly,
yA+yA=y(A+A)
and
yAyA=y(AA).
It follows that
|(x+yA)+(x+yA)|=|2A|
and
|(x+yA)(x+yA)|=|AA|
https://doi.org/10.17993/3cemp.2022.110250.186-196
So the cardinalities of the sum and difference sets of a finite set of integers are invariant under affine
transformations of the set. Easy to see that if
|A|
=
N
, then
|A
+
A|
is bounded above by
N
(
N
+ 1)
/
2
and below by 2N1. The latter occurs when Ais an AP or symmetric.
The set
A
is symmetric with respect to the integer
z
if
A
=
zA
or, equivalently, if
aA
if and
only if zaA. For example, the set {4,6,7,9}is symmetric with z= 13. If Ais symmetric, then
A+A=A+(zA)=z+(AA)
and so
|A
+
A|
=
|AA|
. Equality also holds if
A
is an AP. Examples of equality exists even if
A
is
neither symmetric nor an AP.
A
=
{
0
,
1
,
3
,
4
,
5
,
8
}
is neither symmetric nor an AP but
|A
+
A|
=
|AA|
.
If A={a, b, c}with a<b<cand a+c=2b, then |A+A|=6<7=|AA|.
If
A
=
{
0
,
2
,
3
,
4
,
7
}
then
A
+
A
= [0
,
14]
\{
1
,
12
,
13
}
,
AA
=[
7
,
7]
\ {−
6
,
6
}
and
|A
+
A|
= 12
<
13=|AA|.
This is the typical situation. Since 2+7=7+2but 27=72
It is natural to expect that in any finite set of integers there are always at least as many differences
as sums. There had been a conjecture, often ascribed incorrectly to John Conway, that asserted that
|A+A|≤|AA|for every finite set Aof integers.
This conjecture is false, and a counterexample is the set A={0,2,3,4,7,11,12,14}, for which
A+A= [0,28] \{1,20,27}
AA=[14,14] \ 6,±13}and |A+A|= 26 >25=|AA|.
Given the existence of such aberrant sets, we can ask for the smallest one. The set
A
above satisfies
|A|=8.
Problem 1.
What is the smallest such A?
i.e. find min{|A|:AZand |A+A|>|AA|}?
Note:
Hegarty has proved that this minimum is indeed equal to 8and is affinely equivalent to the set
A={0,2,3,4,7,11,12,14}. One may also ask:
Problem 2.
What is the structure of finite sets satisfying |A+A|>|AA|?
If
A
is a finite set of integers and
m
is a sufficiently large positive integer (for example,
m>
2 max({|a|:aA}), then the set
At=t1
i=0
aimi:aiAfor i=0,1,...,t1
has the property that
|At
+
At|
=
|A
+
A|t
and
|AtAt|
=
|AA|t
. This can be seen as follows: the
elements of
At
+
At
can be thought of as having a base-m expansion with ’digits’ coming from the set
A+A.
It follows that if |A+A|>|AA|, then At+At|>|AtAt|and, moreover,
lim
t→∞ |At+At
|AtAt|= lim
t→∞ |A+A|
|AA|t
=
The sequence of sets
{At}
t=1
is the standard parametrized family of sets with more sums than differences
(called MSTD sets).
Problem 3.
Are there other parametrized families of sets satisfying |A+A|>|AA|?
Even though there exist sets
A
that have more sums than differences, such sets should be rare, and
it must be true with the right way of counting that the vast majority of sets satisfies
|AA|>|A
+
A|
.
Problem 4.
Let
f
(
n
)denote the number sets
A
[0
,n
1] such that
|AA|<|A
+
A|
, and let
f
(
n, k
)denote the
https://doi.org/10.17993/3cemp.2022.110250.186-196
193
3C Empresa. Investigación y pensamiento crítico. ISSN: 2254-3376
Ed. 50 Vol. 11 N.º 2 August - December 2022
number of such sets A[0,n1] with |A|=k. Compute
lim
n→∞
f(n)
2n
and
lim
n→∞
f(n, k)
n
k
What about other functions that count finite sets of nonnegative integers with respect to sums and
differences?
Problem 5.
Prove that
|AA|>|A
+
A|
for almost all sets
A
with respect to other appropriate counting functions.
Binary linear forms:
The problem of sums and differences can be considered a special case of a more
general problem about binary linear forms
f(x, y)=ux +vy
where uand vare nonzero integers. For every finite set Aof integers, let
f(A)={f(a, a):a, aA}.
We are interested in the cardinality of the sets
f
(
A
). For example, the sets associated to the binary
linear forms
s(x, y)=x+y
and
d(x, y)=xy
are the sumset s(A)=A+Aand the difference set d(A)=AA.
To every binary linear form there is a unique normalized binary linear form
f
(
x, y
)=
ux
+
vy
such that
u≥|v|≥1and (u, v)=1.
The natural question is: If
f
(
x, y
)and
g
(
x, y
)are two distinct normalized binary linear forms, do there
exist finite sets
A
and
B
of integers such that
|f
(
A
)
|>|g
(
A
)
|
and
|f
(
B
)
|<|g
(
B
)
|
, and, if so, is there
an algorithm to construct Aand B?
Brooke Orosz gave constructive solutions to this problem in some important cases. For example, she
proved the following: Let u>v1and (u, v)=1, and consider the normalized binary linear forms
f(x, y)=ux +vy and g(x, y)=ux vy.
For u3, the sets
A={0,u
2v2,u
2,u
2+uv}
and
B={0,u
2uv, u2v2,u
2}
satisfy the inequalities
|f(A)|= 14 >13=|g(A)|
https://doi.org/10.17993/3cemp.2022.110250.186-196
and
f(B)=13<14=|g(B)|.
For
u
=2,
v
=1we have
f
(
x, y
)=2
x
+
y
and
g
(
x, y
)=2
xy
. The sets
A
=
{
0
,
3
,
4
,
6
}
and
B={0,4,6,7}satisfy the inequalities |f(A)|= 13 >12 = |g(A)|and |f(B)|= 13 <14 = |g(B)|. The
problem of pairs of binary linear forms has been completely solved by Nathanson, O’Bryant, Orosz,
Ruzsa, and Silva.
Theorem 15.
Let
f
(
x, y
)and
g
(
x, y
)be distinct normalized binary linear forms. There exist finite sets
A, B, C with |C|≥2such that |f(A)|>|g(A)|,|f(B)|<|g(B)|and |f(C)|=|g(C)|.
Problem 6.
Let
f
(
x, y
)and
g
(
x, y
)be distinct normalized binary linear forms. Determine if
|f
(
A
)
|>|g
(
A
)
|
for
most or for almost all finite sets of integers A.
These results should be extended to linear forms in three or more variables.
Problem 7.
Let
f
(
x1,,...,x
n
)=
u1x1
+
...
+
unxn
and
g
(
x1,...,x
n
)=
v1x1
+
...
+
vnxn
be linear forms with
integer coefficients. Does there exist a finite set Aof integers such that |f(A)|>|g(A)|?
Polynomials over finite sets of integers and congruence classes
An integer-valued function is a function
f
(
x1,x
2,...,x
n
)such that if
x1,x
2,...,x
nZ
, then
f(x1,x
2,...,x
n)Z. The binomial polynomial
x
k=x(x1)(x2) ...(xk+ 1)
k!
is integer-valued, and every integer-valued polynomial is a linear combination with integer coefficients
of the polynomials x
k,(George Polya). For any set AZ, we define
f(A)={f(a1,a
2,...,a
n):aiAfor i=1,2,...,n}⊆Z.
Problem 8.
Let
f
(
x1,x
2,...,x
n
)and
g
(
x1,x
2,...,x
n
)be integer-valued polynomials. Determine if there exist
finite sets
A, B, C
of positive integers with
|C|≥
2such that
|f
(
A
)
|>|g
(
A
)
|
,
|f
(
B
)
|<|g
(
B
)
|
and
|f(C)|=|g(C)|. There is a strong form of Problem 8.
Problem 9.
Let f(x1,x
2,...,x
n)and g(x1,x
2,...,x
n)be integer-valued polynomials. Does there exist a sequence
{Ai}
i=1 of finite sets of integers such that
lim
i |f(Ai)|
|g(Ai)|=?
There is also the analogous modular problem. For every polynomial
f
(
x1,x
2,...,x
n
)with integer
coefficients and for every set AZ/mZ, we define
f(A)={f(a1,a
2,...,a
n):aiAfor i=1,2,...,n}⊆Z.
Problem 10.
Let
f
(
x1,x
2,...,x
n
)and
g
(
x1,x
2,...,x
n
)be polynomials with integer coefficients and let
m
2.
Do there exist sets
A, B, C Z/mZ
with
|C|>
1such that
|f
(
A
)
|>|g
(
A
)
|
,
|f
(
B
)
|<|g
(
B
)
|
, and
|f(C)|=|g(C)|.
Problem 11.
Let
f
(
x1,x
2,...,x
n
)and
g
(
x1,x
2,...,x
n
)be polynomials with integer coefficients. Let
M
(
f,g
)denote
https://doi.org/10.17993/3cemp.2022.110250.186-196
3C Empresa. Investigación y pensamiento crítico. ISSN: 2254-3376
Ed. 50 Vol. 11 N.º 2 August - December 2022
194
number of such sets A[0,n1] with |A|=k. Compute
lim
n
f(n)
2n
and
lim
n
f(n, k)
n
k
What about other functions that count finite sets of nonnegative integers with respect to sums and
differences?
Problem 5.
Prove that
|AA|>|A
+
A|
for almost all sets
A
with respect to other appropriate counting functions.
Binary linear forms:
The problem of sums and differences can be considered a special case of a more
general problem about binary linear forms
f(x, y)=ux +vy
where uand vare nonzero integers. For every finite set Aof integers, let
f(A)={f(a, a):a, aA}.
We are interested in the cardinality of the sets
f
(
A
). For example, the sets associated to the binary
linear forms
s(x, y)=x+y
and
d(x, y)=xy
are the sumset s(A)=A+Aand the difference set d(A)=AA.
To every binary linear form there is a unique normalized binary linear form
f
(
x, y
)=
ux
+
vy
such that
u≥|v|≥1and (u, v)=1.
The natural question is: If
f
(
x, y
)and
g
(
x, y
)are two distinct normalized binary linear forms, do there
exist finite sets
A
and
B
of integers such that
|f
(
A
)
|>|g
(
A
)
|
and
|f
(
B
)
|<|g
(
B
)
|
, and, if so, is there
an algorithm to construct Aand B?
Brooke Orosz gave constructive solutions to this problem in some important cases. For example, she
proved the following: Let u>v1and (u, v)=1, and consider the normalized binary linear forms
f(x, y)=ux +vy and g(x, y)=ux vy.
For u3, the sets
A={0,u
2v2,u
2,u
2+uv}
and
B={0,u
2uv, u2v2,u
2}
satisfy the inequalities
|f(A)|= 14 >13=|g(A)|
https://doi.org/10.17993/3cemp.2022.110250.186-196
and
f(B)=13<14=|g(B)|.
For
u
=2,
v
=1we have
f
(
x, y
)=2
x
+
y
and
g
(
x, y
)=2
xy
. The sets
A
=
{
0
,
3
,
4
,
6
}
and
B={0,4,6,7}satisfy the inequalities |f(A)|= 13 >12 = |g(A)|and |f(B)|= 13 <14 = |g(B)|. The
problem of pairs of binary linear forms has been completely solved by Nathanson, O’Bryant, Orosz,
Ruzsa, and Silva.
Theorem 15.
Let
f
(
x, y
)and
g
(
x, y
)be distinct normalized binary linear forms. There exist finite sets
A, B, C with |C|≥2such that |f(A)|>|g(A)|,|f(B)|<|g(B)|and |f(C)|=|g(C)|.
Problem 6.
Let
f
(
x, y
)and
g
(
x, y
)be distinct normalized binary linear forms. Determine if
|f
(
A
)
|>|g
(
A
)
|
for
most or for almost all finite sets of integers A.
These results should be extended to linear forms in three or more variables.
Problem 7.
Let
f
(
x1,,...,x
n
)=
u1x1
+
...
+
unxn
and
g
(
x1,...,x
n
)=
v1x1
+
...
+
vnxn
be linear forms with
integer coefficients. Does there exist a finite set Aof integers such that |f(A)|>|g(A)|?
Polynomials over finite sets of integers and congruence classes
An integer-valued function is a function
f
(
x1,x
2,...,x
n
)such that if
x1,x
2,...,x
nZ
, then
f(x1,x
2,...,x
n)Z. The binomial polynomial
x
k=x(x1)(x2) ...(xk+ 1)
k!
is integer-valued, and every integer-valued polynomial is a linear combination with integer coefficients
of the polynomials x
k,(George Polya). For any set AZ, we define
f(A)={f(a1,a
2,...,a
n):aiAfor i=1,2,...,n}⊆Z.
Problem 8.
Let
f
(
x1,x
2,...,x
n
)and
g
(
x1,x
2,...,x
n
)be integer-valued polynomials. Determine if there exist
finite sets
A, B, C
of positive integers with
|C|≥
2such that
|f
(
A
)
|>|g
(
A
)
|
,
|f
(
B
)
|<|g
(
B
)
|
and
|f(C)|=|g(C)|. There is a strong form of Problem 8.
Problem 9.
Let f(x1,x
2,...,x
n)and g(x1,x
2,...,x
n)be integer-valued polynomials. Does there exist a sequence
{Ai}
i=1 of finite sets of integers such that
lim
i→∞ |f(Ai)|
|g(Ai)|=?
There is also the analogous modular problem. For every polynomial
f
(
x1,x
2,...,x
n
)with integer
coefficients and for every set AZ/mZ, we define
f(A)={f(a1,a
2,...,a
n):aiAfor i=1,2,...,n}⊆Z.
Problem 10.
Let
f
(
x1,x
2,...,x
n
)and
g
(
x1,x
2,...,x
n
)be polynomials with integer coefficients and let
m
2.
Do there exist sets
A, B, C Z/mZ
with
|C|>
1such that
|f
(
A
)
|>|g
(
A
)
|
,
|f
(
B
)
|<|g
(
B
)
|
, and
|f(C)|=|g(C)|.
Problem 11.
Let
f
(
x1,x
2,...,x
n
)and
g
(
x1,x
2,...,x
n
)be polynomials with integer coefficients. Let
M
(
f,g
)denote
https://doi.org/10.17993/3cemp.2022.110250.186-196
195
3C Empresa. Investigación y pensamiento crítico. ISSN: 2254-3376
Ed. 50 Vol. 11 N.º 2 August - December 2022
the set of all integers
m
2such that there exists a finite set
A
of congruence classes modulo
m
such
that |f(A)|>|g(A)|. Compute M(f,g).
Note that if there exists a finite set
A
of integers with
|f
(
A
)
|>|g
(
A
)
|
, then
M
(
f,g
)contains all
sufficiently large integers.
Finally we mention two famous open problems:
1. Erdos:
if
aiN
, such that
i=1 1
ai
=
, then
{ai}
contains arbitrarily long APs. Green-Tao is
a special case since
i=1 1
pi=, where piare all primes.
2. The abc-conjecture:
This says roughly that if a lot of small primes divide
a
and
b
, then only a
few large ones will divide their sum
a
+
b
=
c
. More precisely, we define the
radical
of
nN
as
rad(n)=the product of the distinct prime factors of n. Eg: rad(16)=2, rad(17)=17, rad(18)=6.
Statement
For every
>
0, there exist only finitely many triples (
a, b, c
)of coprime positive integers with
a
+
b
=
c
and c > rad(abc)1+.
This has lot of consequences for mathematics. In particular this also implies the truth of Fermat’s Last
Theorem.
Recently, Shinichi Mochizuki from Japan has claimed a proof of the abc-conjecture. The 500 odd
-page proof is published by the RIMS(Research Institute of Mathematical Sciences), Kyoto, Japan. And
the author is one of editors of the journal. However there is no consensus among mathematicians about
the correctness of the proof. Only future will tell.
REFERENCES
[1] Chu, H. V.
,
McNew, N.
,
Miller, S. J.
,
Xu, V.
and
Zhang S.
(2008). When sets can and cannot
have sum-dominant subsets.J. Integer Seq. 2018. 21(8), 18,8
[2] Granville, A.
, (2007). An introduction to additive combinatorics. In Additive combinatorics.AMS.
2007.Volume 43, 1-27
[3] Nathanson, M. B.
, (2006). Problems in additive number theory, i. arXiv preprint math/0604340,
2006.
[4] Nathanson, M. B.
, (2006). Sets with more sums than differences. arXiv preprint math/0608148,
2006.
[5] Soundararajan„ K., (2010). Additive combinatorics: Winter 2007. Web, 2010.
https://doi.org/10.17993/3cemp.2022.110250.186-196
3C Empresa. Investigación y pensamiento crítico. ISSN: 2254-3376
Ed. 50 Vol. 11 N.º 2 August - December 2022
196